Mathcenter Forum

Mathcenter Forum (https://www.mathcenter.net/forum/index.php)
-   ข้อสอบโอลิมปิก (https://www.mathcenter.net/forum/forumdisplay.php?f=28)
-   -   Road to IMO 2017 to Infinity (https://www.mathcenter.net/forum/showthread.php?t=23527)

Terry Tao 19 ตุลาคม 2016 12:08

Road to IMO 2017 to Infinity
 
1. จงพิสูจน์ว่าสำหรับทุก $a \in \mathbb{Z} $ จะมี $n \in \mathbb{N} $ ที่ทำให้ $2^{2^n}+a$ เป็นจำนวนประกอบบวก

Terry Tao 19 ตุลาคม 2016 12:24

อันนี้สูตร useful cอยู่บนเส้นตรงผ่านzและขนานกับab ก็ต่อเมื่อ (z-c)\(b-a)เป็นจำนวนจริง

Terry Tao 19 ตุลาคม 2016 12:49

อีกสูตร cอยู่บนเส้นตรงผ่านzตั้งฉากกับเส้นตรงab ก็ต่อเมื่อ $\frac{(z-c)}{(b-a)} \in i\mathbb{R}$

Terry Tao 19 ตุลาคม 2016 19:12

Triangle $BCF$ has a right angle at $B$. Let $A$ be the point on line $CF$ such that $FA=FB$ and $F$ lies between $A$ and $C$. Point $D$ is chosen so that $DA=DC$ and $AC$ is the bisector of $\angle{DAB}$. Point $E$ is chosen so that $EA=ED$ and $AD$ is the bisector of $\angle{EAC}$. Let $M$ be the midpoint of $CF$. Let $X$ be the point such that $AMXE$ is a parallelogram. Prove that $BD,FX$ and $ME$ are concurrent.
IMO 2016 problem1 ผมhintให้ครับ1.ไล่มุม 2.วาดวงกลมล้อมรอบBCF 3.อะไรcollinearกันบ้าง 4.หาสี่เหลี่ยมด้านขนานเยอะๆ:wub:

Beatmania 20 ตุลาคม 2016 01:33

เอาจริงๆ เหรอครับ ผมว่าผมสู้พลานุภาพไม่ไหวแน่ๆ 555

แจก NT ให้ข้อนึงครับ

ให้ $m,n$ เป็นจำนวนนับโดยที่ $\phi(5^m-1)=5^n-1$ จงแสดงว่า $gcd(m,n)>1$

ข้อแรกสุด ผมได้แต่แสดงว่ามีอ่ะครับ มีเป็นอนันต์คงจะยาก 555


Terry Tao 20 ตุลาคม 2016 08:26

อ้างอิง:

ข้อความเดิมเขียนโดยคุณ Beatmania (ข้อความที่ 183139)
เอาจริงๆ เหรอครับ ผมว่าผมสู้พลานุภาพไม่ไหวแน่ๆ 555

แจก NT ให้ข้อนึงครับ

ให้ $m,n$ เป็นจำนวนนับโดยที่ $\phi(5^m-1)=5^n-1$ จงแสดงว่า $gcd(m,n)>1$

ข้อแรกสุด ผมได้แต่แสดงว่ามีอ่ะครับ มีเป็นอนันต์คงจะยาก 555


พิสูจน์ว่ามีนั่นแหละครับ ผมจำโจทย์ผิด 5555

Beatmania 20 ตุลาคม 2016 09:47

แจก Set เรขาเผื่อมีคนสนใจครับ

1. ให้ $ABC$ เป็นรูปสามเหลี่ยมด้านไม่เท่าที่มีวงกลม $\Omega $ ล้อมรอบ

เส้นแบ่งครึ่งมุมภายใน $BAC$ ตัดกับ $BC$ และ $\Omega$ ที่จุด E,F ตามลำดับ

วงกลมแนบในสามเหลี่ยม $ABC$ สัมผัสด้าน $BC$ ที่จุด $D$

ให้ $\omega,\Gamma_A$ แทนวงกลมล้อมรอบ $DEF$ และวงกลมแนบนอกตรงข้ามมุม $A$

ถ้าหาก $\omega\cap \Omega=F,T$ และ $\omega\cap \Gamma_A=S_1,S_2$

จงแสดงว่า $A,T,S_1$ อยู่บนเส้นตรงเดียวกัน หรือ $A,T,S_2$ อยู่บนเส้นตรงเดียวกัน (USATST2016 #2)

2. ให้ $ABC$ เป็นรูปสามเหลี่ยมที่มีวงกลม $\Omega $ ล้อมรอบ

ให้ $\omega_A$ แทนวงกลมที่สัมผัสด้าน $AB,AC$ และสัมผัส $\Omega$ แบบภายในที่จุด $T_A$

เรานิยาม $\omega_B,\omega_C,T_B,T_C$ ในทำนองเดียวกัน

จงแสดงว่า $AT_A,BT_B,CT_C$ ตัดกันที่จุดๆ เดียว (Japan MO 2007)

3. เราจะเรียกจุด $P$ ในสามเหลี่ยมมุมแหลมด้านไม่เท่า $ABC$ ว่า "มหัศจรรย์" ถ้าหากสองคล้องกับสมบัติสองข้อต่อไปนี้

1.) ถ้าหากเราลาก $PX\perp BC,PY\perp CA,PZ\perp AB$ โดยที่ $X,Y,Z$ อยู่บนด้าน $BC,CA,AB$

แล้ว $AX,BY,CZ$ ตัดกันที่จุดเดียว

2.) $\angle PAB+\angle PBC+\angle PCA = \angle 90^{\circ}$

จงหาทางเดินจุด (โลคัส) ของจุดมหัศจรรย์ (USATST2016 #6)

4. ให้ $ABC$ เป็นรูปสามเหลี่ยมที่มีวงกลม $\Omega $ ล้อมรอบ

เส้นแบ่งครึ่งมุมภายใน $ABC$ และมุม $BCA$ ตัดด้านตรงข้ามที่จุด $E,F$ ตามลำดับ

เส้นตรง $EF$ ตัดกับ $\Omega$ ที่จุด $M,N$ ให้ $I$ เป็นจุดศูนย์กลางวงกลมแนบใน

จงแสดงว่ารัศมีของวงกลมล้อมรอบ $MIN$ มีขนาดเป็นสองเท่าของรัศมี $\Omega $ (Russia 2006)

5. ให้ $ABC$ เป็นสามเหลี่ยมด้านไม่เท่าที่มี $I$ เป็นจุด ศก วงกลมแนบใน

วงกลมแนบในสัมผัสด้าน $\overline{BC}$, $\overline{CA}$, $\overline{AB}$ ที่จุด $D$, $E$, $F$, ตามลำดับ

ให้ $M$ เป็นจุดกึ่งกลางด้าน $\overline{BC}$, $Q$ เป็นจุดในวงกลมแนบในโดยที่ $\angle AQD = 90^{\circ}$

ให้ $P$ เป็นจุดภายในสามเหลี่ยมและอยู่บนเส้น $AI$ โดยที่ $MD = MP$

จงแสดงว่า $\angle PQE = 90^{\circ}$ หรือ $\angle PQF = 90^{\circ}$ (USATST2015 #1)

Terry Tao 20 ตุลาคม 2016 10:15

เรขาสอบไปครบแล้วครับ ขอเป็น Algebra+IE+FE หน่อยครับ

Beatmania 20 ตุลาคม 2016 10:48

ว่าแล้วว่าต้องเป็นน้องในค่าย 5555 จะจัดให้แบบไม่ผิดหวังแน่นอนครับๆ เอา I ไปก่อนแล้วกัน

1. ให้ $x_1\leq x_2\leq ...\leq x_n$ เป็นจำนวนจริงบวกโดยที่ $\frac{x_1}{1}\geq \frac{x_2}{2}\geq ... \geq \frac{x_n}{n}$ จงแสดงว่า

$$\frac{x_1+x_2+...+x_n}{n\sqrt[n]{x_1x_2...x_n} }\leq \frac{n+1}{2\sqrt[n]{n!} }$$

(China TST 2015)

2. ให้ $a_1,a_2,...,a_n$ เป็นจำนวนจริงบวก จงแสดงว่า

$$\left (\frac{\sum_{j=1}^{n} \left (\prod_{k=1}^{j}a_k \right )^{\frac{1}{j}}}{\sum_{j=1}^{n}a_j} \right )^{\frac{1}{n}}+\frac{\left (\prod_{i=1}^{n}a_i \right )^{\frac{1}{n}}}{\sum_{j=1}^{n} \left (\prod_{k=1}^{j}a_k \right )^{\frac{1}{j}}}\le \frac{n+1}{n}$$

(China TST 2015)

3. จงหาจำนวนจริงบวก $\lambda$ ที่น้อยที่สุดที่มีสมบัติว่า

สำหรับจำนวนเชิงซ้อน $z_1,z_2,z_3$ ที่อยู่ภายในวงกลมหนึ่งหน่วย ถ้าหาก $z_1+z_2+z_3=0$ แล้ว

$$|z_1z_2+z_2z_3+z_3z_1|^2+|z_1z_2z_3|^2<\lambda$$

(China TST 2016)

4. ให้ $z_1,z_2,...,z_n$ เป็นจำนวนเชิงซ้อนและ $r\in (0,1)$โดยที่ $|z_i-1|\leq r$ จงแสดงว่า

$$|\sum_{i = 1}^{n} z_i||\sum_{i = 1}^{n} \frac{1}{z_i} |\geq n^2(1-r^2)$$

(China MO 2015 #1)

5. ให้ $x_1,x_2,...,x_n$ เป็นจำนวนจริงบวกโดยที่ $x_1+x_2+...+x_n=1$ จงแสดงว่า

$$(\sum_{i = 1}^{n} \frac{1}{1-x_i} )(\sum_{1\leq i<j\leq n} x_ix_j)\leq \frac{n}{2} $$

(China Western MO 2015 #3)

Beatmania 20 ตุลาคม 2016 11:24

อันนี้จะเป็น A นะครับ หาโจทย์ยากเหมือนกันนะ 55

1.) พิจารณาระบบสมการที่มี $x_1,x_2,x_3$ เป็นตัวแปรตามด้านล่าง

$$a_{11}x_1+a_{12}x_2+a_{13}x_3=0$$
$$a_{21}x_1+a_{22}x_2+a_{23}x_3=0$$
$$a_{31}x_1+a_{32}x_2+a_{33}x_3=0$$

กำหนดให้ $a_{ij} > 0$ ก็ต่อเมื่อ $i=j$ และผลรวม สปส ในแต่ละสมการมีค่ามากกว่าศูนย์

จงแสดงว่าระบบสมการนี้มีคำตอบแค่หนึ่งชุดคือ $(x_1,x_2,x_3)=(0,0,0)$ (ISL 1965)

2.) กำหนดให้ $a_0<a_1<...$ เป็นลำดับอนันต์ของจำนวนนับ

จงแสดงว่าจะมีจำนวนนับ $n$ เพียงจำนวนเดียวที่ทำให้

$$a_n<\frac{a_0+a_1+...+a_n}{n}<a_{n+1}$$

(IMO 2014 #1)

3.) สำหรับเซต $A,B$ ที่ไม่เป็นเซตว่าง เรากำหนดให้ $A+B=(a+b|a\in A,b\in B)$

เป็นไปได้หรือไม่ที่จะเขียน $\mathbb{Q}$ ในรูปของ $A\cup B\cup C$

โดยที่เซตทั้งสาม disjoint กัน และ $A+B,B+C,C+A$ disjoint กันเช่นกัน

(ISL 2012 A2)

4.) กำหนดให้

$$f(x)=\sum_{i = 1}^{n} a_ix^i,g(x)=\sum_{i = 1}^{n} \frac{a_i}{2^i-1} x^i$$

โดยที่ $n$ เป็นจำนวนนับ $a_1,a_2,...,a_n$ เป็นจำนวนจริงและ $a_n\neq 0$

ถ้าหาก $1,2^{n+1}$ เป็นรากของ $g$ จงแสดงว่า $f$ มีรากที่เป็นบวกและมีค่าน้อยกว่า $2^n$

(USATST 2004)

5.) จงหาพหุนาม $P(x)$ ที่มี สปส เป็นจำนวนจริงทั้งหมดโดยที่

$$P(\sqrt{2}x)=P(x+\sqrt{1-x^2})$$

สำหรับทุกจำนวนจริง $x\in[-1,1]$ (USATSTST 2015 #3)

Terry Tao 20 ตุลาคม 2016 12:15

ขอบคุณครับ

Aquila 20 ตุลาคม 2016 12:33

คุณ Beatmania ไม่ลองส่งพวกชีทแบบแยกเทคนิกให้น้องเขาดูละครับ

อย่าง Geo ก็จะมีหลายๆเทคนิกย่อย รวมทั้งพวก Dark Arts แบบเชิงซ้อน อัดแกนอะไรแบบนี้

ปล.หลังจากเลยช่วงทำโจทย์ส่วนนี้ไป แบบตอนน้องเขาสอบเสร็จแล้ว แบ่งๆกัน :great:

Beatmania 20 ตุลาคม 2016 12:36

มีคนขอ Combi มาครับ 555

1.) พิจารณาส่วนของเส้นตรง $n$ เส้นที่สองเส้นใดๆ ตัดกัน และไม่มีสามเส้นใดที่ตัดกันจุดเดียว

เจฟจะวางกบที่ปลายส่วนของเส้นตรงแต่ละเส้น และวางหันหน้าไปยังปลายอีกด้านหนึ่งขอส่รวนของเส้นตรง

เมื่อเจฟปรบมือ กบจะกระโดดโดยไปยังจุดตัดจุดแรกที่เจอ

เขาต้องการวางกบโดยที่ไม่มีกบสองตัวใดๆ อยู่ในตำแหน่งเดียวกันหลังการปรบมือแต่ละครั้ง

จงแสดงว่า

เจฟจะสามารถวางกบตามที่เขาต้องการได้เสมอเมื่อ $n$ เป็นเลขคี่ และ

เจฟจะไม่สามารถทำตามความต้องการเขาได้ ถ้าหาก $n$ เป็นจำนวนคู่

(IMO2016 #6)

2.) เราจะเรียกรูปหลายเหลี่ยม $P$ ว่า Lattice Polygon ถ้าหากความยาวด้านทุกด้านเป็นจำนวนนับ

และสองด้านที่ติดกันใดๆ จะตั้งฉากกัน

ถ้าหากเราสามารถปู Lattice Polygon P ได้ด้วย S-tetrominoes ได้

จงแสดงว่าไม่ว่าอย่างไรก็ตาม ถ้าหากเราใช้ S-tetrominoes และ Z-tetrominoes ในการปู P

เราจะต้องใช้ Z-tetrominoes เป็นจำนวนคู่ชิ้นเสมอ (ISL 2014 C3)

ปล.ข้อนี้ไปหารูป tetrominoes กันเองนะครับๆ = =" แนบรูปไม่เป็น

3.) มีตารางขนาดอนันต์ แต่ละช่องจะถูกระบายด้วยสีสีหนึ่งจากสีทั้งหมด $1201$ สี

โดยที่มีสมบัติว่า สี่เหลี่ยมผืนผ้าที่มีความยาวรอบรูป 100 ใดๆ ที่คลุมช่องในตารางสนิท

จะไม่มีสองช่องใดๆ ในสี่เหลี่ยมผืนผ้านี้ที่มีสีเดียวกัน

จงแสดงว่า ข้อความด้านบนเป็นจริงสำหรับสี่เหลี่ยมผืนผ้าขนาด $1\times1201$ และ $1201\times 1$

(Balkan MO 2016 #4)

4.) ให้ $x$ เป็นจำนวนอตรรกยะ จงแสดงว่ามีจำนวนเต็ม a,b โดยที่ $\frac{1}{2555}<ax+b<\frac{1}{2012}$

(TMO9 #10 / Special Case of Kronecker's Theorem)

5.) ให้ $X_1, X_2, \ldots, X_{100}$ เป็นสับเซตที่ไม่ใช่เซตว่างและแตกต่างกันของ $S$

สำหรับทุก $i=1,2,...,99$ , $X_i\cap X_{i+1}=\emptyset$ และ $X_i\cup X_{i+1}\neq S$

จงหาค่าที่น้อยที่สุดที่เป็นไปได้ของ $|S|$ (USAMO2016 #1)

Beatmania 20 ตุลาคม 2016 12:39

อ้างอิง:

ข้อความเดิมเขียนโดยคุณ Aquila (ข้อความที่ 183156)
คุณ Beatmania ไม่ลองส่งพวกชีทแบบแยกเทคนิกให้น้องเขาดูละครับ

อย่าง Geo ก็จะมีหลายๆเทคนิกย่อย รวมทั้งพวก Dark Arts แบบเชิงซ้อน อัดแกนอะไรแบบนี้

ปล.หลังจากเลยช่วงทำโจทย์ส่วนนี้ไป แบบตอนน้องเขาสอบเสร็จแล้ว แบ่งๆกัน :great:

จริงๆ ก็ชอบเรขาอ่ะครับ แต่ยังต้องฝึกฝนอีกหน่อยนึงถึงจะเก่ง

พวกเรขาด้านมืดผมแทบไม่ค่อยได้แตะเลยครับ - -" อาจจะแนะนำตรงส่วนนี้ยังไม่ค่อยได้

ปกติผมจะชอบ Synthetic Geometry มากกว่าครับ เหมือนมันเห็นความสวยงามได้มากกว่า

แต่ก็อาจจะมีตรีโกณนิดหน่อยตามความจำเป็น 555

Pitchayut 20 ตุลาคม 2016 17:37

IE แต่ละข้อดูป่าเถื่อนมากเลย แล้วอาจารย์ในค่ายเค้าชอบออกแนว 3-4 ตัวแปรด้วย

Terry Tao 20 ตุลาคม 2016 18:11

คลายเครียดครับ มีP,Qเป็นพหุนามที่ดีกรีมากกว่า1ซึ่งP(Q(x))=(x-1)(x-2)(x-3)...(x-15)หรือไม่

Terry Tao 20 ตุลาคม 2016 18:33

กระจายเทียบสปสก็ได้

Lspeed 20 ตุลาคม 2016 19:35

อ้างอิง:

ข้อความเดิมเขียนโดยคุณ Beatmania (ข้อความที่ 183157)

1.) พิจารณาส่วนของเส้นตรง $n$ เส้นที่สองเส้นใดๆ ตัดกัน และไม่มีสามเส้นใดที่ตัดกันจุดเดียว

เจฟจะวางกบที่ปลายส่วนของเส้นตรงแต่ละเส้น และวางหันหน้าไปยังปลายอีกด้านหนึ่งขอส่รวนของเส้นตรง

เมื่อเจฟปรบมือ กบจะกระโดดโดยไปยังจุดตัดจุดแรกที่เจอ

เขาต้องการวางกบโดยที่ไม่มีกบสองตัวใดๆ อยู่ในตำแหน่งเดียวกันหลังการปรบมือแต่ละครั้ง

จงแสดงว่า

เจฟจะสามารถวางกบตามที่เขาต้องการได้เสมอเมื่อ $n$ เป็นเลขคี่ และ

เจฟจะไม่สามารถทำตามความต้องการเขาได้ ถ้าหาก $n$ เป็นจำนวนคู่

(IMO2016 #6)

กรณีคี่ไม่รู้จะอธิบายยังไงดีครับ :cry:


กขฃคฅฆง 20 ตุลาคม 2016 21:33

อ้างอิง:

ข้อความเดิมเขียนโดยคุณ Lspeed (ข้อความที่ 183167)
กรณีคี่ไม่รู้จะอธิบายยังไงดีครับ :cry:


อ่านยากอยู่นะครับ 55555


Terry Tao 21 ตุลาคม 2016 08:04

Let A, B, C, D, E, and F be the consecutive points of tangency of the small circles with the outer circle and each of small circle tangent to two nearby circle then AD,BE,CF are concurrent.

Terry Tao 21 ตุลาคม 2016 08:06

Hintใช้ceva

Beatmania 21 ตุลาคม 2016 09:57

วันนี้เรียน FE ใช่มั้ยครับ จัดไปครับ

1.) จงหาฟังก์ชัน $f:\mathbb{Z}\rightarrow\mathbb{Z}$ ทั้งหมดที่มีสมบัติว่า

ถ้าหาก $a,b,c$ เป็นจำนวนเต็มที่ $a+b+c=0$ แล้วเราจะได้ว่า

$$f(a)^2+f(b)^2+f(c)^2=2f(a)f(b)+2f(b)f(c)+2f(c)f(a)$$

(IMO 2012 #4)

2.) จงหาฟังก์ชัน $f:\mathbb{Z}\rightarrow\mathbb{Z}$ ทั้งหมดที่

$$f(f(m)+n)+f(m)=f(n)+f(3m)+2014$$

สำหรับทุกจำนวนเต็ม $m,n$ (ISL 2014 A4)

3.) จงหาฟังก์ชัน $f:\mathbb{R}\rightarrow\mathbb{R}$ ทั้งหมดที่

$$f(x+f(x+y))+f(xy)=x+f(x+y)+yf(x)$$

สำหรับทุกจำนวนจริง $x,y$ (IMO 2015 #5)

4.) จงหาฟังก์ชัน $f,g:\mathbb{N}\rightarrow\mathbb{N}$ ทั้งหมดที่มีสมบัติว่า

$$f^{g(n)+1}(n)+g^{f(n)}(n)=f(n+1)-g(n+1)+1$$

สำหรับทุกจำนวนนับ $n$ (ในที่นี้ $f^k(n)$ หมายถึงฟังก์ชันซ้อนกับ $k$ ตัว) (ISL 2011 A4)

5.) ให้ $S$ เป็นสับเซตของจำนวนจริง เราจะกล่าวว่าฟังก์ชัน $f,g:S\rightarrow S$ ว่าเป็น "คู่หู"

ถ้าหากสอดคล้องกันสองเงื่อนไขต่อไปนี้

1.) $f(x)<f(y),g(x)<g(y)$ เมื่อ $x,y\in S$ และ $x<y$

2.) $f(g(g(x)))<g(f(x))$ เมื่อ $x\in S$

จงพิจารณาว่ามีฟังก์ชันที่เป็นคู่หูหรือไม่เมื่อ

1.) $S=\mathbb{N}$

2.) $S=\left\{a-\frac{1}{b}|a,b\in\mathbb{N}\right\} $ (ISL 2008 A3)

Thgx0312555 21 ตุลาคม 2016 16:05

อ้างอิง:

ข้อความเดิมเขียนโดยคุณ Terry Tao (ข้อความที่ 183165)
คลายเครียดครับ มีP,Qเป็นพหุนามที่ดีกรีมากกว่า1ซึ่ง$P(Q(x))=(x-1)(x-2)(x-3)...(x-15)$ หรือไม่

มี solution นึงสวยดีครับ
WLOG ให้ $Q$ เป็น monic จะได้ $P$ เป็น monic ด้วย
ก่อนอื่นจะได้ $\deg [P(x)]=3,\deg [Q(x)]=5$ หรือ $\deg [P(x)]=5,\deg [Q(x)]=3$

กรณี $\deg [P(x)]=3,\deg [Q(x)]=5$ $\quad 1,2,...,15$ เป็นคำตอบของ $P(Q(x))=0$

แต่สมการ $P(x)=0$ มีคำตอบได้อย่างมาก $3$ คำตอบ สมมติคำตอบเหล่านั้นคือ $a,b,c$
ต่อมาสมการ $Q(x)=a,Q(x)=b,Q(x)=c$ มีคำตอบได้อย่างมากสมการละ $5$ คำตอบ

แต่เราทราบว่าสามสมการนี้ต้องมีคำตอบรวมกัน $15$ คำตอบ จึงทำให้แต่ละสมการมี $5$ คำตอบพอดี

เราจึงได้ระบบสมการต่อไปนี้
$Q(x)=(x-r_1)(x-r_2)\cdots (x-r_5)+a$
$Q(x)=(x-r_6)(x-r_7)\cdots (x-r_{10})+b$
$Q(x)=(x-r_{11})(x-r_{12})\cdots (x-r_{15})+c$
เมื่อ $r_1,r_2,...,r_{15}$ เป็นการเรียงสับเปลี่ยนของ $1,2,...,15$

ดังนั้นโดยผลบวกราก $r_1+r_2+\cdots+r_5=r_6+r_7+\cdots+r_{10}=r_{11}+r_{12}+\cdots+r_{15}$
และ
$\displaystyle \sum_{1\le i < j \le 5} r_ir_j=\sum_{6\le i < j \le {10}} r_ir_j=\sum_{{11}\le i < j \le {15}} r_ir_j$

จะได้ $r^2_1+r^2_2+\cdots+r^2_5=r^2_6+r^2_7+\cdots+r^2_{10}=r^2_{11}+r^2_{12}+\cdots+r^2_{15} = \dfrac{1}{3}(1^2+2^2+\cdots+15^2) = \dfrac{1240}{3}$ จึงเกิดข้อขัดแย้ง

อีกกรณีนึงสามารถทำเช่นเดียวกัน แต่จะมาเปลี่ยนตรงท้ายจะกลายเป็น
$r_1+r_2+r_3=r_4+r_5+r_6=\cdots=r_{13}+r_{14}+r_{15}=24$
$r^2_1+r^2_2+r^2_3=r^2_4+r^2_5+r^2_6=\cdots=r^2_{13}+r^2_{14}+r^2_{15}=248$
WLOG $r_1=15$ จะได้ $r_2 \ge 5$ หรือ $r_3 \ge 5$ ทำให้ $r_1^2+r_2^2+r_3^2 > 248$ เกิดข้อขัดแย้ง

ปล.1 อย่าพึ่งรีบ hint สิครับ
ปล.2 อาจารย์ในค่ายค่อนข้างชอบอสมการที่มีค่ากึ่งกลางนะ

ไว้ว่างๆจะหาโจทย์มาให้น้องในค่ายทำเหมือนกัน แต่ช่วงนี้ก็ไม่ค่อยได้ทำโจทย์ละครับ

Thgx0312555 21 ตุลาคม 2016 16:35

ให้ข้อคลาสสิกไปข้อนึงดีกว่า
ในลำดับที่ประกอบด้วยจำนวน $ab+1$ จำนวนซึ่งแตกต่างกันทั้งหมด จงพิสูจน์ว่าจะมีลำดับย่อยซึ่งมีสมาชิก $a+1$ ตัวและเป็นลำดับเพิ่ม หรือมีสมาชิก $b+1$ ตัวและเป็นลำดับลด

นิยาม: ลำดับย่อยคือลำดับที่เกิดจาก delete สมาชิกในลำดับเริ่มต้นออกไปบางตัว โดยไม่เปลี่ยนตำแหน่งของตัวที่เหลือ
https://en.wikipedia.org/wiki/Subsequence

Pitchayut 21 ตุลาคม 2016 17:48

วิธีผมเอง ไม่รู้มีใครทำแบบนี้ป่าว

ให้ $s_1,s_2,s_3,...,s_{ab+1}$

สมมติขัดแย้งว่า ทุกๆลำดับย่อยเพิ่มแท้ มีความยาวไม่เกิน $a$ และ ทุกๆลำดับย่อยลดแท้ มีความยาวไม่เกิน $b$

จะทาสีตัวเลขทุกตัว โดยให้ $f(n)$ แทนสีของ $s_n$ โดยทาสีตามขั้นตอนต่อไปนี้

1. ทาสี $s_1$ ด้วยสีที่ 1 จากนั้นหาตัวแรกที่น้อยกว่า $s_1$ ให้เป็นตัวที่ $t_{1,2}$)

2. ทำนองเดียวกัน หาตัวแรกที่น้อยกว่า $s_{t_{1,2}}$ ให้เป็นตัวที่ $t_{1,3}$ จากนั้นทำไปเรื่อยๆจนจบลำดับ

3. ทาสี $s_{t_{1,k}}$ ด้วยสีที่ 1 ทุกๆ $k$

4. ดำเนินการเช่นนี้ไปเรื่อยๆ โดยเริ่มจากตัวแรกที่ไม่ถูกทาสี และให้สีเป็น $2,3,4,...$ ไปเรื่อยๆ

สังเกตว่าจะมีช่องที่ทาสีที่ $i$ อยู่ไม่เกิน $b$ ช่อง (เพราะมิฉะนั้นจะเกิดลำดับลดความยาว $b+1$)

และมีสีไม่เกิน $a$ สี เพราะตัวแรกที่ทาสีที่ $i$ สำหรับ $i=1,2,3,...$ จะทำให้เกิดลำดับเพิ่ม

เพราะฉะนั้น จำนวนพจน์ต้องมีไม่เกิน $ab$ พจน์ ซึ่งเป็นข้อขัดแย้ง

Lspeed 21 ตุลาคม 2016 20:51

อ้างอิง:

ข้อความเดิมเขียนโดยคุณ Beatmania (ข้อความที่ 183157)

4.) ให้ $x$ เป็นจำนวนอตรรกยะ จงแสดงว่ามีจำนวนเต็ม a,b โดยที่ $\frac{1}{2555}<ax+b<\frac{1}{2012}$

(TMO9 #10 / Special Case of Kronecker's Theorem)

โดยส่วนตัวผมว่าข้อนี้ยากนะครับ :):):)


Terry Tao 21 ตุลาคม 2016 22:30

ข้อนี้สวยงามครับ ให้f: (0,infinity)->(0,infinity) นิยามโดยf(xf(x)+f(y))=y+f(x)^2 จงหาf(x)ทั้งหมดที่เป็นไปได้

Terry Tao 21 ตุลาคม 2016 22:33

Hint:ถ้าไม่มีf(0)หาf(1)แทนแต่ก่อนจะพบค่าf(1)จะพบความมหัศจรรย์ที่นำไปสู่คำตอบ

กขฃคฅฆง 21 ตุลาคม 2016 22:49

อย่ารีบ hint สิ 555555

Lspeed 21 ตุลาคม 2016 23:49

คุณ Terry tao ไม่ต้องรีบใบ้ขนาดนั้นก็ได้คร้าบ

Thgx0312555 22 ตุลาคม 2016 00:18

ผมทำวิธีเดียวกันนิแหละ แต่ถ้าทำแบบนี้มันจะติดปัญหาตรงนี้

อ้างอิง:

ข้อความเดิมเขียนโดยคุณ Pitchayut (ข้อความที่ 183175)
วิธีผมเอง ไม่รู้มีใครทำแบบนี้ป่าว

ให้ $s_1,s_2,s_3,...,s_{ab+1}$

สมมติขัดแย้งว่า ทุกๆลำดับย่อยเพิ่มแท้ มีความยาวไม่เกิน $a$ และ ทุกๆลำดับย่อยลดแท้ มีความยาวไม่เกิน $b$

จะทาสีตัวเลขทุกตัว โดยให้ $f(n)$ แทนสีของ $s_n$ โดยทาสีตามขั้นตอนต่อไปนี้

1. ทาสี $s_1$ ด้วยสีที่ 1 จากนั้นหาตัวแรกที่น้อยกว่า $s_1$ ให้เป็นตัวที่ $t_{1,2}$)

2. ทำนองเดียวกัน หาตัวแรกที่น้อยกว่า $s_{t_{1,2}}$ ให้เป็นตัวที่ $t_{1,3}$ จากนั้นทำไปเรื่อยๆจนจบลำดับ

3. ทาสี $s_{t_{1,k}}$ ด้วยสีที่ 1 ทุกๆ $k$

4. ดำเนินการเช่นนี้ไปเรื่อยๆ โดยเริ่มจากตัวแรกที่ไม่ถูกทาสี และให้สีเป็น $2,3,4,...$ ไปเรื่อยๆ

สังเกตว่าจะมีช่องที่ทาสีที่ $i$ อยู่ไม่เกิน $b$ ช่อง (เพราะมิฉะนั้นจะเกิดลำดับลดความยาว $b+1$)

และมีสีไม่เกิน $a$ สี เพราะตัวแรกที่ทาสีที่ $i$ สำหรับ $i=1,2,3,...$ จะทำให้เกิดลำดับเพิ่ม

เพราะฉะนั้น จำนวนพจน์ต้องมีไม่เกิน $ab$ พจน์ ซึ่งเป็นข้อขัดแย้ง

จริงเหรอครับ

ปล. ข้อนี้มี alternate solution ที่สวยจริงๆ อยู่ในลิงค์ wiki ที่แปะลิงค์ไปแล้ว

Beatmania 22 ตุลาคม 2016 00:57

จริงๆ ผมชอบ NT สุด จะไม่ปล่อย NT มันก็กระไรอยู่ 555

1.) ให้ $k,n$ เป็นจำนวนนับโดยที่

$$\underbrace{\phi(\phi(...\phi(}_{k} n)...)=1$$

จงแสดงว่า $n\leq 3^k$ (USATSTST 2016 #4)

2.) ให้ $\sqrt{3}=1.b_1b_2..._{(2)}$ เป็นการเขียน $\sqrt{3}$ ในรูปเลขฐานสอง

จงแสดงว่าสำหรับทุกจำนวนนับ $n$ มีอย่างน้อยหนึ่งจำนวนใน $b_n,b_{n+1},...,b_{2n}$ ที่มีค่าเป็น $1$

(USATST2016 #4)

3.) ให้ $a,b$ เป็นจำนวนนับโดยที่ $a!+b!|a!b!$ จงแสดงว่า $3a\geq 2b+2$ (ISL 2015 N2)

4.) สำหรับจำนวนนับ $n$ เรานิยามให้ $D_n=\left\{d-\frac{n}{d}:d|n,d<\sqrt{n}\right\} $

จงแสดงว่ามีจำนวนนับ $n_1,n_2,...,n_{2016}$ ที่ทำให้

$$|D_{n_1}\cap D_{n_2}\cap ...\cap D_{n_{2016}}|>1$$

(Modified from China MO 2015 #2)

5.) จงหาพหุนาม $P(x)\in\mathbb{Z}[x]$ ทั้งหมดที่มีสมบัติว่ามีจำนวนเต็ม $n$ เป็นอนันต์ที่ $P(n+P(n))$ เป็นจำนวนเฉพาะ

(Canada MO 2016 #3)

ที่ผมลงไปมี 30 ข้อ ทำได้ซัก 15+ ข้อก็คงจะได้ สสวท. 2 ไม่ยากแล้วครับๆ :)

Pitchayut 22 ตุลาคม 2016 18:26

FE ข้อ 1,3 ได้แล้ว (ขอไม่ลง sol เพราะเหมือนกับ sol ตามท้องตลาด)

NT ข้อ 5 ลองแสดงดูก่อนว่า $P(n)\mid P(n+P(n))$ ที่เหลือก็แค่ไล่ไปเรื่อยๆ

Lspeed 23 ตุลาคม 2016 07:36

พี่ๆครับ พอจะมีโจทย์แนวinvarian,monovarianที่น่าสนใจบ้างไหมครับ?

Pitchayut 23 ตุลาคม 2016 17:18

NT ข้อ 3 unseen แต่ง่ายดีครับ ใช้ความรู้ไม่เกินค่าย 1

สำหรับคนที่อยากฝึก Lifting The Exponent Lemma

1. จงพิสูจน์ว่าไม่มี $(b,m,n)\in\mathbb{N}^3$ ที่ทำให้ $b> 1$ และ $b^m-1$ กับ $b^n-1$ มีเซตของตัวประกอบเฉพาะเป็นเซตเดียวกัน

2. จงหา $(a,m,n)\in\mathbb{N}^3$ ทั้งหมดที่ทำให้ $a^m+1\mid (a+1)^n$

อยากได้ Hint ข้อไหนบอกได้นะครับ

Thgx0312555 24 ตุลาคม 2016 11:38

Hint NT3

Pitchayut 24 ตุลาคม 2016 17:28

ผมก็ทำประมาณนั้นแหละครับคุณ Thgx

แล้วก็โจทย์ LTE 2 ข้อ ที่ให้ไปพยายามอย่าใช้ Zsigmondy นะครับ

Beatmania 25 ตุลาคม 2016 23:53

ผมใจดีแจก Hint ให้สำหรับโจทย์ชุดของผมนะครับ :)


ว่างแล้วเดี๋ยวมาพิมพ์ต่อครับ

Lspeed 29 ตุลาคม 2016 21:23

อ้างอิง:

ข้อความเดิมเขียนโดยคุณ Pitchayut (ข้อความที่ 183228)
สำหรับคนที่อยากฝึก Lifting The Exponent Lemma

1. จงพิสูจน์ว่าไม่มี $(b,m,n)\in\mathbb{N}^3$ ที่ทำให้ $b> 1$ และ $b^m-1$ กับ $b^n-1$ มีเซตของตัวประกอบเฉพาะเป็นเซตเดียวกัน

แก้ไม่ออกครับยอม:please:

แต่ว่าผมงงอย่างนึง $(3,1,2)$ มันได้ $3^1-1 = 2$ และ $3^2-1 = 8$ ซึ่งเซตตปก.เฉพาะคือ $\left\{\,\right. 2\left.\,\right\}$ ทั้งคู่หนิครับ ผมเข้าใจโจทย์ผิดไปตรงไหนเปล่าครับ

Beatmania 30 ตุลาคม 2016 14:14

อ้างอิง:

ข้อความเดิมเขียนโดยคุณ Lspeed (ข้อความที่ 183274)
แก้ไม่ออกครับยอม:please:

แต่ว่าผมงงอย่างนึง $(3,1,2)$ มันได้ $3^1-1 = 2$ และ $3^2-1 = 8$ ซึ่งเซตตปก.เฉพาะคือ $\left\{\,\right. 2\left.\,\right\}$ ทั้งคู่หนิครับ ผมเข้าใจโจทย์ผิดไปตรงไหนเปล่าครับ

คุณ Lspeed ถูกแล้วครับ จริงๆ ต้องมีเงื่อนไขที่ว่า $b+1$ ไม่เป็นกำลังของสองด้วยครับ

ปล. ไม่มีใครทำเรขาของผมจริงๆ เหรอครับ T_T ผมว่าประมาณนี้เหมาะสำหรับ สสวท ค่าย 1 เลยนะครับๆ


เวลาที่แสดงทั้งหมด เป็นเวลาที่ประเทศไทย (GMT +7) ขณะนี้เป็นเวลา 17:20

Powered by vBulletin® Copyright ©2000 - 2024, Jelsoft Enterprises Ltd.
Modified by Jetsada Karnpracha